Top-Tier GMAT Score

Quiz for students needing a top-tier GMAT score

Below, you'll find 15 official (retired) GMAT questions that are in the 650 to 800 range of difficulty.

As you may already know, the GMAT is a timed test. So, to get an accurate idea of your present GMAT skills, give yourself 30 minutes to complete the quiz.

Once you've answered all 15 questions, check the answer key at the very bottom of the page to determine how many questions you answered correctly. Beneath the answer key, you'll find advice regarding how to interpret your results. 

Important: If you know very little about the GMAT and the skills it tests, you might want to first watch this brief video overview of the GMAT.

NOTE: About 40% of all questions in the Quantitative Reasoning section of the GMAT are Data Sufficiency (DS) questions, which are unique to GMAT. This question type can be quite confusing at first. So, before attempting the questions below, take a few minutes to familiarize yourself with this question type by watching some videos in the free Data Sufficiency lesson module.

 

START OF QUIZ (30 minutes) 

 

Quantitative Reasoning – Problem Solving Questions (select the best answer)

1) One night a certain hotel rented 3/4 of its rooms. including 2/3 of their air conditioned rooms. if 3/5 of its rooms were air conditioned, what percent of the rooms that were not rented were air conditioned?

A. 20

B. 33 1/3

C. 35

D. 40

E. 80

 

2) What is the sum of odd integers from 35 to 85, inclusive?

A) 1,560

B) 1,500

C) 1,240

D) 1,120

E) 1,100

 

3) At a certain fruit stand, the price of each apple is 40 cents and the price of each orange is 60 cents. Mary selects a total of 10 apples and oranges from the fruit stand, and the average (arithmetic mean) price of the 10 pieces of fruit is 56 cents. How many oranges must Mary put back so that the average price of the pieces of fruit that she keeps is 52 cents?

(A) 1

(B) 2

(C) 3

(D) 4

(E) 5



4) If k is an integer and 2 < k < 7, for how many different values of k is there a triangle with sides of lengths 2, 7, and k?

(A) one

(B) two

(C) three

(D) four

(E) five

 

5) A car traveling at a certain constant speed takes 2 seconds longer to travel 1 kilometer than it would take to travel 1 kilometer at 75 kilometers per hour. At what speed, in kilometers per hour, is the car traveling?

A. 71.5

B. 72

C. 72.5

D. 73

E. 73.5

 

Quantitative Reasoning – Data Sufficiency Questions

Note: If you haven’t already done so, you should learn more about Data Sufficiency questions before attempting the next 3 questions (otherwise, these questions will be a complete mystery!)

 

6) On a certain sight-seeing tour, the ratio of the number of women to the number of children was 5 to 2. What was the number of men on the sight-seeing tour?

(1) On the sight-seeing tour, the ratio of the number of children to the number of men was 5 to 11.

(2) The number of women on the sight-seeing tour was less than 30.

Directions

This data sufficiency problem consists of a question and two statements, labeled (1) and (2), in which certain data are given. You have to decide whether the data given in the statements are sufficient for answering the question. Using the data given in the statements, plus your knowledge of mathematics and everyday facts (such as the number of days in July or the meaning of the word counterclockwise), you must indicate whether:

(A) Statement (1) ALONE is sufficient, but statement (2) alone is not sufficient to answer the question asked.

(B) Statement (2) ALONE is sufficient, but statement (1) alone is not sufficient to answer the question asked.

(C) BOTH statements (1) and (2) TOGETHER are sufficient to answer the question asked, but NEITHER statement ALONE is sufficient to answer the question asked.

(D) EACH statement ALONE is sufficient to answer the question asked.

(E) Statements (1) and (2) TOGETHER are NOT sufficient to answer the question asked, and additional data specific to the problem are needed.

 

7) What is the value of x if x³ < x²?

(1) –2 < x < 2

(2) x is an integer greater than –2.

Directions

This data sufficiency problem consists of a question and two statements, labeled (1) and (2), in which certain data are given. You have to decide whether the data given in the statements are sufficient for answering the question. Using the data given in the statements, plus your knowledge of mathematics and everyday facts (such as the number of days in July or the meaning of the word counterclockwise), you must indicate whether:

(A) Statement (1) ALONE is sufficient, but statement (2) alone is not sufficient to answer the question asked.

(B) Statement (2) ALONE is sufficient, but statement (1) alone is not sufficient to answer the question asked.

(C) BOTH statements (1) and (2) TOGETHER are sufficient to answer the question asked, but NEITHER statement ALONE is sufficient to answer the question asked.

(D) EACH statement ALONE is sufficient to answer the question asked.

(E) Statements (1) and (2) TOGETHER are NOT sufficient to answer the question asked, and additional data specific to the problem are needed.

 

8) Joanna bought only $0.15 stamps and $0.29 stamps. How many $0.15 stamps did she buy?

(1) She bought $4.40 worth of stamps.

(2) She bought an equal number of $0.15 stamps and $0.29 stamps.

Directions

This data sufficiency problem consists of a question and two statements, labeled (1) and (2), in which certain data are given. You have to decide whether the data given in the statements are sufficient for answering the question. Using the data given in the statements, plus your knowledge of mathematics and everyday facts (such as the number of days in July or the meaning of the word counterclockwise), you must indicate whether:

(A) Statement (1) ALONE is sufficient, but statement (2) alone is not sufficient to answer the question asked.

(B) Statement (2) ALONE is sufficient, but statement (1) alone is not sufficient to answer the question asked.

(C) BOTH statements (1) and (2) TOGETHER are sufficient to answer the question asked, but NEITHER statement ALONE is sufficient to answer the question asked.

(D) EACH statement ALONE is sufficient to answer the question asked.

(E) Statements (1) and (2) TOGETHER are NOT sufficient to answer the question asked, and additional data specific to the problem are needed.

 

Verbal Reasoning – Critical Reasoning

The directions for all Critical Reasoning questions are the same: Select the best of the answer choices given.

 

9) The average hourly wage of television assemblers in Vernland has long been significantly lower than that in neighboring Borodia. Since Borodia dropped all tariffs on Vernlandian televisions three years ago, the number of televisions sold annually in Borodia has not changed. However, recent statistics show a drop in the number of television assemblers in Borodia. Therefore, updated trade statistics will probably indicate that the number of televisions Borodia imports annually from Vernland has increased.

Which of the following is an assumption on which the argument depends?

A. The number of television assemblers in Vernland has increased by at least as much as the number of television assemblers in Borodia has decreased.

B. Televisions assembled in Vernland have features that televisions assembled in Borodia do not have.

C. The average number of hours it takes a Borodian television assembler to assemble a television has not decreased significantly during the past three years.

D. The number of televisions assembled annually in Vernland has increased significantly during the past three years.

E. The difference between the hourly wage of television assemblers in Vernland and the hourly wage of television assemblers in Borodia is likely to decrease in the next few years.

 

10) Images from ground-based telescopes are invariably distorted by the Earth's atmosphere. Orbiting space telescopes, however, operating above Earth's atmosphere, should provide superbly detailed images. Therefore, ground-based telescopes will soon become obsolete for advanced astronomical research purposes.

Which of the following statements, if true, would cast the most doubt on the conclusion drawn above?

(A) An orbiting space telescope due to be launched this year is far behind schedule and over budget, whereas the largest ground-based telescope was both within budget and on schedule.

(B) Ground-based telescopes located on mountain summits are not subject to the kinds of atmospheric distortion which, at low altitudes, make stars appear to twinkle.

(C) By careful choice of observatory location, it is possible for large-aperture telescopes to avoid most of the kind of wind turbulence that can distort image quality.

(D) When large-aperture telescopes are located at high altitudes near the equator, they permit the best Earth-based observations of the center of the Milky Way Galaxy, a prime target of astronomical research.

(E) Detailed spectral analyses, upon which astronomers rely for determining the chemical composition and evolutionary history of stars, require telescopes with more light-gathering capacity than space telescopes can provide.

 

11) A prominent investor who holds a large stake in the Burton Tool company has recently claimed that the company is mismanaged, citing as evidence the company's failure to slow production in response to a recent rise in its inventory of finished products. It is doubtful whether an investor's sniping at management can ever be anything other than counterproductive, but in this case it is clearly not justified. It is true that an increased inventory of finished products often indicates that production is outstripping demand, but in Burton's case it indicates no such thing. Rather, the increase in inventory is entirely attributable to products that have already been assigned to orders received from customers.

In the argument given, the two boldfaced portions play which of the following roles?

(A) The first states the position that the argument as a whole opposes; the second provides evidence to undermine the support for the position being opposed.

(B) The first states the position that the argument as a whole opposes; the second is evidence that has been used to support the position being opposed.

(C) The first states the position that the argument as a whole opposes; the second states the conclusion of the argument as a whole.

(D) The first is evidence that has been used to support a position that the argument as a whole opposes; the second provides information to undermine the force of that evidence.

(E) The first is evidence that has been used to support a position that the argument as a whole opposes; the second states the conclusion of the argument as a whole.

 

12) Which of the following most logically completes the argument?

Lyme disease is caused by a bacterium transmitted to people by deer ticks. Deer ticks are almost never born infected with the bacterium, but they can pick it up from feeding on an infected host in any of the tick's three successive stage of development: c. Once infected, a tick remains infected through any subsequent developmental stages, but since a tick feeds on only one host in each stage, it follows that ______________

A. People can be infected only by being bitten by a tick that is either in larval stage or in the nymph stage

B. People cannot be infected by a tick that picked up the Lyme disease bacterium in the adult stage

C. It is not possible for a person to be infected by a tick that is in the nymph stage or by a tick that was infected with the bacterium while in the nymph stage

D. People infected with the Lyme disease bacterium cannot transmit bacterium to adult deer ticks

E. Only deer ticks that are adults can transmit the Lyme disease bacterium to people.

 

Verbal Reasoning – Sentence Correction

The directions for Sentence Correction questions are always the same. They are as follows:

This question presents a sentence, part of which or all of which is underlined. Beneath the sentence you will find five ways of phrasing the underlined part. The first of these repeats the original; the other four are different. If you think the original is best, choose the first answer; otherwise choose one of the others.



13) In addition to having more protein than wheat does, the protein in rice is higher quality than that in wheat, with more of the amino acids essential to the human diet.

(A) the protein in rice is higher quality than that in

(B) rice has protein of higher quality than that in

(C) the protein in rice is higher in quality than it is in

(D) rice protein is higher in quality than it is in

(E) rice has a protein higher in quality than

 

14) According to a 1996 survey by the National Association of College and University Business Officers, more than three times as many independent institutions of higher education charge tuition and fees of under $8,000 a year than those that charge over $16,000

A. than those that charge

B. than are charging

C. than to charge

D. as charge

E. as those charging

 

15) The largest of all the planets, not only is Jupiter three times so massive as Saturn, the next larger planet, but also possesses four of the largest satellites, or moons, in our solar system.

A: not only is Jupiter three times so massive as Saturn, the next larger

B: not only is Jupiter three times as massive as Saturn, the next largest

C: Jupiter, not only three times as massive as Saturn, the next largest

D: Jupiter not only is three times as massive as Saturn, the next largest

E: Jupiter is not only three times so massive as Saturn, the next larger

 



END OF QUIZ: Scroll down to see answer key (and links to step-by-step solutions).

 

 

 

 

 

 

 

 

 

 

 

Answer Key

  1. Answer: E   (Brent’s solution)
  2. Answer: A   (Brent’s solution)
  3. Answer: E   (Brent’s solution)
  4. Answer: A   (Brent’s solution)
  5. Answer: B   (Brent’s solution)
  6. Answer: C   (Brent’s solution)
  7. Answer: B   (Brent’s solution)
  8. Answer: A   (Brent’s solution)
  9. Answer: C   (Brent’s solution)
  10. Answer: E   (Brent’s solution)
  11. Answer: A   (Brent’s solution)
  12. Answer: B   (Brent’s solution)
  13. Answer: B   (Brent’s solution)
  14. Answer: D   (Brent’s solution)
  15. Answer: D   (Brent's solution)

 

Interpreting your results

If you need a GMAT score between 650 and 800, and you correctly answered:

  • 12 or more questions (in under 30 minutes), then it's likely that you already have the skills needed to achieve your target score. To be certain, however, it’s a good idea to take an official full-length practice test, and see how you do.
  • 8 to 11 questions, then it’s possible that can reach your target score within 1 month of preparing (provided you work hard during that time).
  • fewer than 8 questions, then you will likely need more than 1 month to prepare.



     

Study Guide

The step-by-step Study Guide will help direct your studies and ensure that you cover everything that the GMAT tests.

Free “Question of the Day” emails!